- PowerScore Staff
- Posts: 5972
- Joined: Mar 25, 2011
- Tue Feb 06, 2018 2:07 pm
#43575
Complete Question Explanation
(The complete setup for this game can be found here: lsat/viewtopic.php?t=8478)
The correct answer choice is (C)
All three templates are possible if X is not in P. Thus, P is either filled by F, H and Y, or F, I and W.
Answer choice (A) is incorrect because F cannot be in S.
Answer choice (B) is incorrect because if H is in S then W must be in P (Templates #2 and #3) for this question.
Answer choice (C) is the correct answer choice (Template #1).
Answer choice (D) is incorrect because G cannot be in P.
Answer choice (E) is incorrect because if H is in P it must be with F and Y, not W.
(The complete setup for this game can be found here: lsat/viewtopic.php?t=8478)
The correct answer choice is (C)
All three templates are possible if X is not in P. Thus, P is either filled by F, H and Y, or F, I and W.
Answer choice (A) is incorrect because F cannot be in S.
Answer choice (B) is incorrect because if H is in S then W must be in P (Templates #2 and #3) for this question.
Answer choice (C) is the correct answer choice (Template #1).
Answer choice (D) is incorrect because G cannot be in P.
Answer choice (E) is incorrect because if H is in P it must be with F and Y, not W.
Dave Killoran
PowerScore Test Preparation
Follow me on X/Twitter at http://twitter.com/DaveKilloran
My LSAT Articles: http://blog.powerscore.com/lsat/author/dave-killoran
PowerScore Podcast: http://www.powerscore.com/lsat/podcast/
PowerScore Test Preparation
Follow me on X/Twitter at http://twitter.com/DaveKilloran
My LSAT Articles: http://blog.powerscore.com/lsat/author/dave-killoran
PowerScore Podcast: http://www.powerscore.com/lsat/podcast/